Why Does Angular Velocity Reach 24 rad/s in This Motion Equation Problem?

In summary, the conversation discusses a problem setup with a slender uniform rigid arm in space and equations of motion for its inertial velocity and angular velocity. The applied moment is given and integrated in MATLAB to produce a plot. Questions are raised about the magnitude of the angular velocity and the rapid increase in the inertial velocity. The conversation also mentions using polar coordinates for the calculations.
  • #1
boeing_737
12
0
Hi,

I am having a little bit of conceptual trouble with this problem and would appreciate your help. The problem setup is given in the figure. Let's say we have a slender uniform rigid arm(mass m, length l) in space, with a coordinate system [itex]B[/itex] attached to the left end of the arm as shown. C is the center of mass of the arm. We have a moment [itex]M_{z_b}[/itex] acting about the [itex]\hat{z}_{b}[/itex] axis.

physics_forums_fig.png


Let [itex](u,v,w)[/itex] and [itex](p,q,r)[/itex] be the inertial velocity and inertial angular velocity vectors expressed in [itex]B[/itex]. I get the scalar equations of motion as (assuming that the angular velocity is only along [itex]\hat{z}_b[/itex]):

[itex]m \dot{u} - \frac{ml}{2} r^2 = F_{x_b}[/itex]
[itex]m \dot{v} + \frac{ml}{2} \dot{r} = F_{y_b}[/itex]
[itex]m \dot{w} = F_{z_b}[/itex]

[itex]0 = M_{x_b}[/itex]
[itex]-\frac{ml}{2} \dot{w} = M_{y_b}[/itex]
[itex]\frac{ml^2}{3} \dot{r} + \frac{ml}{2} \dot{v} = M_{z_b}[/itex]

The applied moment is given as : [itex]M_{z_b}(t) = 160 \left(1 - \cos \left(\frac{2 \pi t}{15} \right) \right)[/itex]. For [itex]t > 15, M_{z_b} = 0[/itex]. See figure below :
physics_forums_fig2.jpg


Integrating these equations using MATLAB's ode45, I get the following plot :
physics_forums_fig3.jpg


From the above figure :
1) There is only one component of angular velocity (yaw rate) which is as expected. But is the magnitude correct (ie should it reach 24 rad/s)?

2) I am not able to figure out what's going on with u. Why is it increasing so rapidly?

Any help would be really appreciated.

yogesh
 
Last edited:
Physics news on Phys.org
  • #2
1. it's your math.
The total change in momentum is the area of the force-time graph ... so you can check.
I don't know why you are not doing this in polar coordinates.

2. I imagine because the moment is quite high.
Have you got any reason to expect u to increase less rapidly?
What sort of value were you expecting and why?
 

What is an equation of motion?

An equation of motion is a mathematical expression that describes the motion of an object. It typically includes information about the object's position, velocity, and acceleration over time.

What are the three main equations of motion?

The three main equations of motion are:

  • Position equation: x = x0 + v0t + ½at2
  • Velocity equation: v = v0 + at
  • Acceleration equation: v2 = v02 + 2a(x-x0)

These equations can be used to solve for an unknown variable (such as acceleration) given the other variables.

How do I solve for acceleration using equations of motion?

To solve for acceleration, you will need to use one or more of the three main equations of motion. Identify which variables (position, velocity, and acceleration) are given and which are unknown, and then choose the appropriate equation(s) to use. Once you have the equation(s) set up, simply plug in the known values and solve for the unknown variable.

What is the difference between uniform and non-uniform motion?

Uniform motion is when an object moves with a constant velocity (speed and direction) over time. Non-uniform motion is when an object's velocity changes over time, either by changing speed or direction.

Can equations of motion be used for any type of motion?

Equations of motion can be used for any type of motion, as long as the motion can be described as linear (i.e. in a straight line). They may not be applicable for more complex motions, such as circular or projectile motion.

Similar threads

Replies
2
Views
863
Replies
3
Views
494
Replies
3
Views
863
  • Mechanics
Replies
24
Views
983
Replies
4
Views
2K
Replies
1
Views
1K
Replies
2
Views
4K
  • Introductory Physics Homework Help
Replies
2
Views
232
  • Introductory Physics Homework Help
Replies
24
Views
262
  • Advanced Physics Homework Help
Replies
11
Views
1K
Back
Top